Can anyone answer this?

Can Anyone Answer This?

Answers

Answer 1

Answer:

  ∠AMB = 103°

Step-by-step explanation:

The sum of angles in ∆AMB is ...

  (∠A)/2 +(∠B)/2 + ∠AMB = 180°

  ∠AMB = 180° -(∠A +∠B)/2 = 180° -(58° +96°)/2

  ∠AMB = 103°


Related Questions

4. Suppose you have a map of Charlotte where 2
inches equals 4 miles. On the map, the distance
from your house to school if 3.5 inches. How
many miles is this?

Answers

convert inches to miles .

Hey everybody! Can anybody help me out with this algebra problem? I would really appreciate it if you can!

The following relation is a variation. Which description best represents the relation?​

Answers

Answer:

x and y are directly related because y = -3x

Step-by-step explanation:

Cost in dollars Books Delivered $27.25 4 $39.25 6 $75.25 12 $93.25 15 Mr. Peterson's Book Store delivers to customers after they have purchased. The costs are shown above in the table a. What is the cost per book? b. What is the delivery fee? Solve each proportion

Helpppp pleaseeeee ​

Answers

Answer:

  a) cost per book: $6

  b) delivery fee: $3.25

Step-by-step explanation:

a)

If we assume the cost per book and the delivery fee are constant, then we can find the cost of 2 books by subtracting the cost for 4 books from the cost for 6 books

  cost per book = (cost for 6 books - cost for 4 books)/(6 -4)

  = ($39.25 -27.25)/2 = $12/2 = $6 . . . . . cost per book

__

b)

The delivery charge will be the difference between the delivered cost and the cost of the books. For 4 books, that is ...

  delivery charge = $27.25 -4(6.00) = $3.25

_____

Additional comment

The relationship between delivered cost and number of books is not proportional. There are no proportions to solve in this problem.

The _______ of an observation is determined by how far the values of the independent variables are from their means.

Answers

Answer:

The leverage of an observation is determined by how far the values of the independent variables are from their means.

A force of 56N is required to keep a spring stretched 4m from the equilibrium position. How much work in Joules is done to stretch the spring 8m from equilibrium? Round your answer to the nearest hundredth if necessary.

Answers

Please check the attached images for the answer. Answer with full explanation is screenshoted. Plz refer.

Thanks ☺️

Hope it helps you.

Sharon’s turtle escaped from her backyard sometime in the last few hours. According to her calculations, the farthest the turtle could have gone is 4 blocks down the road in either direction. If Sharon lives on the 112th block of town, which equation can be used to find the block numbers that represent the farthest distance that the turtle may be?

Answers

The equation u can use is subtraction or division to get your answer if u use subtraction u will get an answer of 108 block of the town and if u use division u would get 28th block of the town

The circumference of a circle is 240 meters. What is the approximate
radius of the circle? (Use 3.14 for 7)
A) 9 meters
B) 38 meters
C) 76 meters
D) 153 meters

Answers

answer is ‘38 meters’ B
Answer: B
Example: 340 divid by 3.14 you get 76
76 divid by 2 you get 38

A bookshelf contains 6 German books, 4 Spanish books and 7 French books. Each book is different from one another. How many different arrangements can be done of these books if

Answers

Answer:

17 different arrangements

Step-by-step explanation:

6+4+7=17

Sam has 3 bottles of water. Together Sam and Dave have at most 12 bottles of water. How many bottles of water can Dave have?

Inequality-

ANSWER- ​

Answers

Answer: 9

Step-by-step explanation:

Answer:

4

Step-by-step explanation:

If you divide 12 and 3 you will equal 4

How many tens equal 6 thousands

Answers

600 is the correct answer
the answer is= 600 tens

Please help!

The black graph is the graph of
y = f(x). Choose the equation for the
red graph.

Answers

Answer:

it would be c because its moved 2x up

Step-by-step explanation:

I have 4 groups . After I dived Ed them into groups there were 6 groups 0f 4 and 1 group of 3. How many students were in the class ? Show all your work.

Answers

Answer: 27 students

Step-by-step explanation: 6 x 4 = 24

24 + 3 = 27

Answer:

27 Students in the class

Step-by-step explanation:

First we have to find out how many people are in the 6 groups of 4. 6x4=24

And of course 1 group of 3 people. Add them together to get 27 people.

24+3=27

if there was a group of ten people the probability of them being born on the same day of the same year is 1, true or false and why

Answers

Answer:

True

Step-by-step explanation:

For instance, twins are born on the same day but imagine ten people instead of two being born on the same day.

What integer that best describes the statement "the stock rose 8 points"?

A.-8
B.15
C.-15
D.+8​​

Answers

Answer:

D.+8​​

Step-by-step explanation:

"The stock rose 8 points."

The word rose indicates that the number will be positive.

Hope this helps :)

Answer:

D

Step-by-step explanation:

hope it helps

Adam And Eve had 32 apple in 8 barrels how many would they have in all?

Answers

Answer:

Adam and Eve will have 4 apples in each barrel

Step-by-step explanation:

32/8

= 4

What should be done to both sides of the equation in order to solve -5m = -40?


A. Multiply by -5.
B. Divide by -5.
C. Multiply by -40.
D. Divide by -40.

Answers

Answer
B. Divide by -5

Answer:

B.Divide by -5

Step-by-step explanation:

-5m= -40

divide bothe sides by -5

m= -40/-5

m= 8

Hello there hello hello

Answers

Answer:

(-4,-7)

Step-by-step explanation:

The process of a reflection across the y axis is simply just switching the x value in (x,y) to a negative. So, (4,-7) and then make 4, a -4. So the resulting image is (-4,-7)

Answer:

Ah hello!

Step-by-step explanation:

Jerry left a ​$5.55 tip on a meal that cost ​$37. What percent of the​ meal's cost was the​ tip? What was the total cost of the meal including the​ tip?

Answers

If 10% of 37 is 3.70 and 20% of 7.40, then it has to be something inbetween.
the tip was 15% of the meal cost making the total cost $42.55

How to find answer to: 8% of ? = 20

Answers

Answer:

You have to do 8/100 * 20

Step-by-step explanation:

Steps to solve "what percent is 8 of 20?" If you are using a calculator, simply enter 8÷20×100 which will give you 40 as the answer.

Find the average rate of change of the function from x=1 to x=2.



f(x)=−10/x^2


Enter the exact answer.



Average rate of change =

Answers

Answer:

Average rate of change = [tex]\frac{15}{2}[/tex]

Step-by-step explanation:

For a closed interval [tex][a,b][/tex] in a function, the average rate of change is [tex]m=\frac{f(b)-f(a)}{b-a}[/tex].

Thus, given our interval of [tex][1,2][/tex] from the directions, we determine that [tex]f(b)=f(2)=-\frac{10}{(2)^2}=-\frac{10}{4}=-\frac{5}{2}[/tex] and [tex]f(a)=f(1)=-\frac{10}{(1)^2}=-\frac{10}{1}=-10[/tex].

Hence, [tex]m=\frac{f(b)-f(a)}{b-a}=\frac{(-\frac{5}{2})-(-10) }{2-1}=\frac{\frac{15}{2}}{1}=\frac{15}{2}[/tex].

In conclusion, the average rate of change of the function from x=1 to x=2 is [tex]\frac{15}{2}[/tex].

If Clare takes out a mortgage for 22 years at an interest rate of 3% and her monthly repayments are $984, what is the principal loan amount?

Answers

Using simple interest, it is found that the principal loan amount was of $156,491.6.

The amount of money after t years in simple interest is modeled by:

[tex]A(t) = A(0)(1 + rt)[/tex]

In which:

A(0) is the initial amount, which is also called principal. r is the interest rate, as a decimal.

In this problem, she paid $984 monthly for 22 years, hence:

[tex]t = 22, A(t) = 984(22)(12) = 259776[/tex]

Interest rate of 3%, hence [tex]r = 0.03[/tex]

Then:

[tex]A(t) = A(0)(1 + rt)[/tex]

[tex]259776 = A(0)[1 + 0.03(22)][/tex]

[tex]A(0) = \frac{259776}{1 + 0.03(22)}[/tex]

[tex]A(0) = 156491.6[/tex]

The principal loan amount was of $156,491.6.

A similar problem is given at https://brainly.com/question/13176347

HELP ME ASAP!!

Solve the system.
2x - 5y = -8
2x - 5y = 12
Which statement is true?
A. The solution is (1,2).
B. The system has infinitely many solutions.
O C. The system has no solution.

Answers

No answers at all! (C) 2x-5y cannot equal two things at once.

After three exams, Jenny has a mean score of 74. With only one exam remaining in the class, what is the
minimum score she will need on that exam to have an overall mean of 80?

Answers

She will need at least a 98

The minimum score Jenny need on that exam to have an overall mean of 80 is 98.

What is mean?

In statistics, the mean refers to the average of a set of values. The mean can be computed in a number of ways, including the simple arithmetic mean (add up the numbers and divide the total by the number of observations).

Given that, after three exams, Jenny has a mean score of 74.

Let the minimum score of the one exam remaining be x

Now, (3×74+x)/4 =80

⇒ 222+x=320

⇒ x=320-222

⇒ x=98

Therefore, the minimum score Jenny need on that exam to have an overall mean of 80 is 98.

To learn more about an arithmetic mean visit:

https://brainly.com/question/15196910.

#SPJ2

Given m \| nm∥n, find the value of x. m n t (4x+4)° (2x-16)° Answer:

Answers

Answer:

4x+4)° (2x-16)

Step-by-step explanation:

(4x+4)° (2x-16)°

Compare:
5/14 11/18
.
>
<
=

Answers

Answer:

<

Step-by-step explanation:

hope this helps <33

At gym A is 76.50 for 8.5 hours. At gym B cost is 10.50 for 3/4 hours which plan of offers the best deal

Answers

The better deal is gym a.

Tell me if you want step by step explanation!

Hope this helped. :)
Gym A is the correct answer.

Suppose that a risk-free investment will make three future payments of $250 in 1 year, $250 in 2 years, and $250 in 3 years. Instructions: Round your answers to 2 decimal places. a. If the Federal Reserve has set the risk-free interest rate at 4 percent, what is the proper current price of this investment? $ b. What is the price of this investment if the Federal Reserve raises the risk-free interest rate to 6 percent? $

Answers

Answer:

la théorie des plaques est

The total present value will be $693.75

The total present value will be $668.23

What is Future Value?

Future value is the value of an asset or investment as of some future date. In other words, the future value is the sum of money that an investment will be worth, assuming a certain rate of return, after a specific amount of time (interest rate).

First, i= 4/100 = 0.04

n= 1, 2, 3

Amount= $250

So, Present Value = 250/ (1 + 0.04) +  250/ (1 + 0.04)² +  250/ (1 + 0.04)³

=  250/ 1.04 + 250/ 1.04² + 250/ 1.04³

= 240. 38 + 231.13 + 222.24

= $693.75

So, The total present value will be $693.75

Second,  i= 6/100 = 0.06

n= 1, 2, 3

Amount= $250

So, Present Value = 250/ (1 + 0.06) +  250/ (1 + 0.06)² +  250/ (1 + 0.06)³

=  250/ 1.06 + 250/ 1.06² + 250/ 1.06³

= 235. 84 +  222.49 + 209.90

= $668.23

So, The total present value will be $668.23

Learn more about Future Value here:

https://brainly.com/question/14860893

#SPJ2

no link no bot right please

Answers

Answer:

2143.57 cm^3

Step-by-step explanation:

the formula for the volume of a sphere is (4/3)pi*r^3. plug 8cm in for r to get (4/3)*3.14*8^3=2143.57

Carpet sells for $3 per square foot and will cost you $675 to recarpet your rectangular room. If your room is 15 feet long, how many feet wide is it?

Answers

675/3=225
225/15=15
It’s a15x15 room

OMG HELP RN

On a typical night at a restaurant, the patio lights use 202.64 kilowatt-hours of electricity and the courtyard lights use h kilowatt-hours of electricity.

Which of the following expressions represents the total number of kilowatt-hours of electricity the patio and courtyard lights use?
A.
202.64 - h
B.
202.64 ÷ h
C.
202.64 × h
D.
202.64 + h

Answers

Answer:

C would be the answer

Hope this helps!

Other Questions
which general staff member directs management of all 14. At a local fruit stand, Luisa spends $5.25 for 2 pounds of strawberries. At this rate, how much can she expect to pay for 3.5 pounds of strawberries? (round to the nearest hundredth) Proportion Answer HELPPPPP ASAP!!!! Im on a limit Vocal percussion on a whole 'notha levelComing from my mindVocal percussion on a whole 'notha levelComing from my mind An applied force accelerates a 4.00 kg block to an initial velocity of 11 m/s across a rough horizontal surface, in the positive x direction. As the block reaches 11 m/s, the applied force is removed. The block then slows to 1.5 m/s at a distance of 4.00 m beyond where the applied force was removed. Determine the magnitude and the direction of the non-conservative force acting on the box as it slides. 100 POINTS! Select the correct answer.Which system of equations is represented by this graph? What is the primary purpose of a human community? Darren believed in saving money and followed a strict monthly budget. He wanted to invest his savings in a safe option that paid more than his savings account and he was content with earning a lower, consistent amount of money. He did not want to worry about losing any money with market changes. Which type of investment below should Darren consider? CDs Business startups New stock offerings Junk bonds Why did Ted Slavin start Essential Biologicals? why would an ecosystem with only a few different types of organisms have a less complex food web than an ecosystem with many different of organisms how much does it cost to have a float in the macy parade (7+7b)+(-2+8c)+(1+5d) since her firms sales have increased from 3.5 to 4 percent, tanya has determined she would set her share of communication expenses at 4 percent of sales, which is an example of the use of the ________ method. please answer fully/////////////// What is the equation, in slope-intercept form, that relates the cost compared to the miles traveled for a Green Cab? FIND THE ERROR Fern is finding the slope of the line that passes through (-2, 8) and (4. 6). Determine in which step she made an error. Ex 1\4-2\5plese help and thank you x2 +x + 36Help me lol The French military leader montcalm was defeated in what is now maine.True or False? A family of four drove to Disney World: It took 12 hours to get there. Parking was $15. The total cost of parking and admission was $195. How much was each ticket?